You are on page 1of 76

The Field of Pediatrics

Question . 1. For which age group are unintentional injuries not the leading cause of death?

1-4 yr

5-9 yr

Under 1 yr
Explanation: (See Chapter 1 in Nelson Textbook of Pediatrics, 17th edition.)

10-14 yr
15-19 yr

Question . 2. Which of the following statements is false?

Children (0-18 yr) make up about 50% of the population


Explanation: (See Chapter 1 in Nelson Textbook of Pediatrics, 17th edition.)

In 1999, over 25% of children under 18 yr were living with one parent

Infant mortality in the United States was 6.9/1000 live births in 2000

Postnatal infant mortality in the United States was 2.3/1000 live births in 2000

In 1998, 18% of all children in the United States lived in families with income
below the poverty level ([dollar]16,530 for a family of four persons)

Question . 3. All of the following statements about ethics are true except:

The age at which a competent patient may legally exercise voluntary informed
consent for medical care varies from state to state

The decision for a DNAR order does not imply a decision to withhold other
aspects of providing medical treatment

The decision to withhold life-sustaining medical treatment implies an intent or


choice to hasten a child's death
Explanation: (See Chapter 2 in Nelson Textbook of Pediatrics, 17th edition.)

Medically beneficial treatment can be withheld from a severely disabled infant


who is permanently unconscious

Teenagers may not be able to support themselves but still be competent to


consent to health care
Question . 4. Which of the following statements is false?

The World Health Organization (WHO) has a program called Integrated


Management of Childhood Illness (IMCI), which focuses on a single medical
condition providing all levels of care.
Explanation: (See Chapter 4 in Nelson Textbook of Pediatrics, 17th edition.)

About one-third of children younger than 15 yr in developing countries either


have or are at risk for impaired learning.

About 18,000 children are adopted into American families from abroad each year.

Most disasters occur in developing countries and are due to natural events such
as hurricanes.

Economic conditions and family resources drive health care decision-making.

Question . 5. What is an optimal schedule for health supervision visits of a child during the first year of
life?

1 mo, 6 mo, 12 mo

1 mo, 3 mo, 6 mo, 12 mo

1 wk, 1 mo, 3 mo, 6 mo, 12 mo

1 wk, 1 mo, 2 mo, 6 mo, 9 mo, 12 mo

Prenatal, 1 wk, 1 mo, 2 mo, 4 mo, 6 mo, 9 mo, 12 mo


Explanation: (See Chapter 5 in Nelson Textbook of Pediatrics, 17th edition.)

Question . 6. Which of the following statements is true?

Children are at the forefront of societal and cultural change related to health and
well-being

Ethnicity encompasses culture


Explanation: (See Chapter 3 in Nelson Textbook of Pediatrics, 17th edition.)

Variability within any cultural or ethnic group often exceeds that between groups

The use of complementary and alternative medicine occurs in educated and


uneducated affluent groups

Children are active shapers of their cultural environment rather than passive
recipients of socialization
Question . 7. Which of the following statements is false?

Ninety percent of children in the early 21st century are born into the developing
world

The mortality rate of children under 5 yr worldwide is 80-90/1000

Between 1980 and 2000 the percentage of children immunized in developing


countries stayed constant
Explanation: (See Chapter 4 in Nelson Textbook of Pediatrics, 17th edition.)

Approximately one third of children under 15 yr in developing countries either


have or are at risk for impaired learning

The Integrated Management of Childhood Illness (IMCI) program uses any point of
contact as an opportunity to assess the overall health of the child and family in
developing countries

Question . 8. A 9-mo-old girl has repeated night awakenings after going to sleep and has been unable to
return to sleep without breastfeeding. This behavior has been present for the past month, prior to which
she slept the entire night without waking or requiring middle-of-the-night feedings. Her growth and
development are normal. She breastfeeds 6-7 times/day and eats a variety of solid foods. Of the following,
which is the most likely explanation for this behavior?

Diminishing breast milk production

Increased growth velocity

Separation anxiety
Explanation: (See Chapter 5 in Nelson Textbook of Pediatrics, 17th edition.)

Feeding schedule: trained night feeder

Nightmares

Question . 9. A 2-yr-old boy refuses to hold his mother's hand when walking across the parking lot of a
shopping mall. He attempts to run away from her but she quickly grabs his hand before he darts out in
front of a car. She is exasperated and scared by his actions.
Part A Of the following, which technique is most effective in addressing the behavior?

Verbal reprimand

Spanking

Scolding

Time-out
Explanation: (See Chapter 5 in Nelson Textbook of Pediatrics, 17th edition.)

Slapping his hand


Question . Part B To be effective, time-outs should:

Be accompanied by verbal reprimands

Be accompanied by punishment

Last at least 5-10 min

Last 1 min per year of age


Explanation: (See Chapter 5 in Nelson Textbook of Pediatrics, 17th edition.)

Be done at a point in time removed from the incident

Question . 10. The sensitivity of carefully performed observation, history, and physical examination in the
child with an acute illness is:

70%

60%

90%
Explanation: (See Chapter 6 in Nelson Textbook of Pediatrics, 17th edition.)

100%

50%

Question . 11. Most observation data that pediatricians gather during an acute illness should focus on
assessing which of the following?

Eye behavior

Gross motor ability

Fine motor ability

Work of breathing

Response to stimuli
th
Explanation: (See Chapter 6 in Nelson Textbook of Pediatrics, 17 edition.)
Question . 12. In the acutely ill child younger than 36 mo, which of the following components of the clinical
assessment is best done on the examination table?

Eye examination

Abdominal examination
Explanation: (See Chapter 6 in Nelson Textbook of Pediatrics, 17th edition.)

Pulmonary auscultation

Range and ease of extremity motion

Cardiac auscultation

Question . 13. Paradoxic irritability is a finding from observation that is helpful in diagnosing:

Pneumonia

Urinary tract infection

Septic arthritis

Meningitis
Explanation: (See Chapter 6 in Nelson Textbook of Pediatrics, 17th edition.)

Bacteremia

Question . 14. The white blood cell count and differential is most helpful as a screening test in which of the
following entities?

Meningitis

Bacteremia
Explanation: (See Chapter 6 in Nelson Textbook of Pediatrics, 17th edition.)

Pneumonia

Urinary tract infection

Septic arthritis
Question . 15. Generally, a competent adult can make a decision to accept or reject a physician's
recommendation regardless of the impact on his or her health. A parent must make decisions about a
child's health care based on a physician's recommendations. The range of acceptable decisions that a
parent can make, however, is restricted. Of the following, which is the most important factor for a
pediatrician to consider when evaluating a parent's decision about the health care for a 7-yr-old child?

The expressed wishes of the child

What a reasonable parent would do in that situation

The pediatric clinician's independent professional obligation to act in a child's


best interest
Explanation: (See Chapter 2 in Nelson Textbook of Pediatrics, 17th edition.)

The ability of the parent to pay for the recommended medical care

The opinions of other pediatric clinicians

Question . 16. At times, the assent of a child to a proposed medical intervention is necessary to assure
appropriate treatment. In seeking a child's assent, which of the following is not recommended?

Help a child understand his or her condition

Provide guidance to the child on what is best for him or her


Explanation: (See Chapter 2 in Nelson Textbook of Pediatrics, 17th edition.)

Tell a child what he or she can expect

Assess a child's understanding and whether a child feels pressured to assent

Solicit a child's willingness to participate

Question . 17. A parent has made a decision not to pursue further treatment for a child's condition, given
the medical recommendation that there are no further treatments that offer any hope of extending the
child's life. Of the following treatments, which is not considered an appropriate option to either withdraw or
withhold?

Artificial hydration and nutrition

Adequate sedation and analgesia


Explanation: (See Chapter 2 in Nelson Textbook of Pediatrics, 17th edition.)

Antibiotics

Mechanical ventilation

Inotropic medications
Question . 18. Many health care institutions have established ethics committees made up of a diverse
membership. Of the following, which is generally not a function of the ethics committee?

Drafting a review of institutional policy

Educating health care professionals, patients, and families

Case consulting

Responding to parental requests for ethical advice

Triage of scarce hospital resources


Explanation: (See Chapter 2 in Nelson Textbook of Pediatrics, 17th edition.)

Question . 19. As many adults choose not to be tested for late-onset disorders, we cannot assume that a
child would want or would benefit from similar testing. Genetic testing of childhood and adolescents for
late-onset disorders is generally inappropriate, except under specific circumstances. Of the following,
which is the best reason for genetic testing of late-onset disorders in childhood?

The parents need to make decisions concerning long-term savings for


educational expenses

Scarce family resources require triaging educational support to that child without
the genetic condition

Such testing will result in interventions that have been shown to reduce morbidity
and mortality when initiated in childhood
Explanation: (See Chapter 2 in Nelson Textbook of Pediatrics, 17th edition.)

The parents want to seek suitable marriage partners and need to know the child's
carrier state

The family health insurance is set to expire, and such testing will thus not be
available in the future
Growth and Development, Set 1

Question . 1. Behavioral states in the newborn period include:

Quiet and active sleep

Drowsy and alert states

Fussy and crying states

All of the above


Explanation: Newborn infants have six characteristic organizational
behavioral states. (See Chapter 9 in Nelson Textbook of Pediatrics, 17th
ed.)

None of the above

Question . 2. The best formula to approximate average weight (kg) for a 4-year-old is:

Age (years) 7 - 5/2

Age (years) 2 + 8
Explanation: The formula in choice A is used for 7- to 12-yr-old children;
C, for 3- to 12-mo-old infants; D, for 1- to 6-yr-olds in pounds; and E, for
7- to 12-yr-olds in pounds. (See Chapter 15 in Nelson Textbook of
Pediatrics, 17th ed.)

Age (months) + 9/2

(Age [years] 5 + 17)

(Age [years] 7 + 5)

Question . 3. A normal infant may cry for up to 3 hr/day during the developmental peak time of
this behavior. This peak is typically at age:

2 wk

4 wk

6 mo

6 wk
Explanation: Crying may or may not be in response to obvious stimuli
(e.g., need for a diaper change). (See Chapter 10 in Nelson Textbook of
Pediatrics, 17th ed.)

4 mo
Question . 4. The best feeding protocol for a temperamentally irregular infant is:

A fixed schedule

One based on the parents' schedule

Every 1-2 hr

One based on demand


Explanation: Demand feedings prevent periods of hunger and episodes
of being fed while not being hungry for a child with an irregular rhythm.
(See Chapter 11 in Nelson Textbook of Pediatrics, 17th ed.)

60 min for each feeding

Question . 5. Object permanence is not present at age 2 mo. The response to a ball dropped in
front of the child is:

Staring momentarily at the spot the ball was dropped from


Explanation: "Out of sight, out of mind" is the characteristic response of
a 2-mo-old. Object permanence appears at approximately 8 mo of age.
This is also called object constancy. (See Chapter 10 in Nelson Textbook
of Pediatrics, 17th ed.)

Eyes lowering as the ball descends

Crying when the ball hits the ground

Smiling at the game of hide-and-seek

None of the above

Question . 6. The ability to manipulate small objects with the pincer grasp is usually noted at
age:

0-2 mo

3-5 mo

6-7 mo

8-9 mo
Explanation: The pincer grasp, which is noted at 8 to 9 mo, along with
increasing mobility, enables an infant to explore the environment. (See
Chapter 10 in Nelson Textbook of Pediatrics, 17th ed.)

10-12 mo
Question . 7. The probable age of a developmentally normal child who is just able to sit without
support, can transfer objects from hand to hand, and speaks in a monosyllabic babble is:

3 mo

4 mo

9 mo

6 mo
Explanation: Each pediatrician should learn key developmental
milestones such as these. (See Chapter 10 in Nelson Textbook of
Pediatrics, 17th ed.)

11 mo

Question . 8. Transitional objects include:

Training underwear

Shoes without laces

Cups with a special drinking spout

Blankets and teddy bears


Explanation: Transitional objects help toddlers (18-24 mo) cope with
separation (e.g., at nighttime for sleep, with baby sitter, at daycare). (See
Chapter 11 in Nelson Textbook of Pediatrics, 17th ed.)

None of the above

Question . 9. Handedness is usually determined by age:

2-4 mo

6-12 mo

15-18 mo

20-24 mo

36-48 mo
Explanation: Handedness should not be attempted to be modified
because this leads to frustration. After age 4 years, a spontaneous
change in handedness should lead to the suspicion of a central nervous
system lesion. (See Chapter 12 in Nelson Textbook of Pediatrics, 17th
ed.)
Question . 10. The best approach for parents to help a preschool child overcome monster fears
is to:

Rationalize that monsters do not exist

Read books that do not have monsters in them

Have the pediatrician explain that monsters are make-believe

Use "great power" like monster spray to keep monsters away


Explanation: It is impossible to rationalize away a preschool child's fear
of monsters. (See Chapter 12 in Nelson Textbook of Pediatrics, 17th ed.)

None of the above

Question . 11. A mother brings her 6-mo-old circumcised boy to you for a "sick" visit. You saw
the child 2 wk previously for health maintenance, including a DTP immunization, and the child
appeared well. The mother's complaint is that the baby is waking up every night and is fussy
during the day, especially when she leaves him. The child's history is otherwise normal, and
physical examination reveals no problems. The most appropriate approach to management is
to:

Perform urinalysis and obtain a complete blood count to rule out urinary
tract infection

Request that the mother feed the infant more

Reassure the mother that the behavior is normal and will pass in time
Explanation: Waking up at night (if in fact the baby had already slept
through part of the night) at 6 to 8 mo is common behavior. Whether this
is related to separation anxiety or something else (teething?) is not clear
(edentulous babies wake up, too). Choice A would be highly unlikely,
because 6-mo-old circumcised boys who have grown normally rarely
contract urinary tract infections. Choice B would be unnecessary
because increased food intake does not relieve night fussiness. D is
wrong because DTP reactions occur 4-36 hr after the shot, not 2 wk. (See
Chapter 10 in Nelson Textbook of Pediatrics, 17th ed.)

Reassure the mother that the behavior will pass because it is a reaction
to the DTP shot

Question . 12. The biopsychosocial model of development, when applied to the child's height,
includes all of the following except:

Genetic endowment

Personal eating habits

Access to food
Parents' beliefs

Differences between breast milk and formula


Explanation: In general, all other socioeconomic factors held constant,
both means of infant feeding are equally effective. Nonetheless, in less
advantaged environments, breast milk has specific biologic advantages.
The biopsychosocial model helps combine the nature versus nurture
aspects of previous theories of development. (See Chapter 7 in Nelson
Textbook of Pediatrics, 17th ed.)

Question . 13. All of the following statements regarding a child's temperament are true except:

Temperament is absolute and stable throughout the life span


Explanation: Temperament is only moderately stable over time, and a
mellow 2-yr-old does not always equate to a mellow 22-yr-old. (See
Chapter 7 in Nelson Textbook of Pediatrics, 17th ed.)

Biology influences temperament

It is a pattern of the child's responses

It is relatively resistant to parents' attempts to modify

It helps parents understand the child's behavior without guilt

Question . 14. The parents of a 3-yr-old girl report that "she ran before she walked," "she is
never hungry at the same time," and "she goes from toy to toy." This child is best described as:

Autistic

Having a specific temperament


Explanation: The seven characteristics of temperament have a wide
range of responsiveness; these characteristics are activity level,
rhythmicity, approach and withdrawal, adaptability, threshold of
responsiveness, intensity of reaction, quality of mood, distractibility,
attention span, and persistence. (See Chapter 7 in Nelson Textbook of
Pediatrics, 17th ed.)

Having attention deficit hyperactivity disorder

Having developmental pervasive disorder

Being deaf
Question . 15. The visual acuity of a newborn permits recognition of an object held at a distance
of:

1-2 inches

8-12 inches
Explanation: The near-sighted neonate has a fixed focal length of 8-12
inches. The newborn also has a visual preference for faces. (See Chapter
9 in Nelson Textbook of Pediatrics, 17th ed.)

15-24 inches

24-30 inches

30-36 inches

Question . 16. A newborn infant spends about 40 min with the mother but then falls asleep and
does not respond to the mother's voice. Which of the following statements regarding this lack of
activity is true?

It is suggestive of sepsis

It is suggestive of sedation

It is normal
Explanation: This normal sleep after a 40-min period of social interaction
is a great time to continue the bonding process. (See Chapter 9 in Nelson
Textbook of Pediatrics, 17th ed.)

It represents a seizure

It is due to apnea

Question . 17. The six behavioral states of the neonate include all of the following except:

Quiet sleep

Active sleep

Drowsiness

Alertness

Colic
Explanation: Colic is not a neonatal state and does not even occur
during the neonatal period. (See Chapter 9 in Nelson Textbook of
Pediatrics, 17th ed.)

Fussiness

Crying
Question . 18. The best formula for approximating average weight in kilograms for a 9-mo-old is

Age (mo) + 9/2


Explanation: B is best for 9-mo-olds in pounds. (See Chapter 10 in
Nelson Textbook of Pediatrics, 17th ed.)

Age (mo) + 11

Age (yr) 2 + 8

Age (yr) 5 + 17

Age (yr) 7 + 5

Question . 19. The best formula for approximating average height in centimeters for a 4-yr-old is

Age (yr) 2.5 + 30

Age (yr) 6 + 77
Explanation: A is in inches. (See Chapter 10 in Nelson Textbook of
Pediatrics, 17th ed.)

Age (yr) 7 + 5

Age (yr) 5 + 7

Age (yr) 2.5 - 6

Question . 20. All of the following statements regarding growth in the first month of life are true
except:

Weight may decrease 10% in the first week

Weight should equal or exceed birthweight by 2 wk

Once gaining weight, the infant should gain 30 g/day

Preterm infants take longer to regain birthweight

The high fat content of colostrum enhances weight gain in the first week
of life
Explanation: Colostrum has a high protein content. Mature milk has a
higher fat and lower protein content than those of colostrum. (See
Chapter 10 in Nelson Textbook of Pediatrics, 17th ed.)
Question . 21. Crying in the first 2 mo of life is characterized by all of the following except:

Teething
Explanation: Teething does not occur until approximately 5-6 mo. (See
Chapter 10 in Nelson Textbook of Pediatrics, 17th ed.)

Peaking at 6 wk

Peaking at a total of 3 hr/day

Crying in response to obvious stimuli

Crying when no stimulus is obvious

Question . 22. The probable age of a child who scribbles, walks alone, speaks one real word,
and pretends to drink from a cup is:

8 mo

13 mo
Explanation: These milestones correspond respectively to visual-motor
coordination, exploration labeling, and symbolic thought. (See Chapter
10 in Nelson Textbook of Pediatrics, 17th ed.)

16 mo

20 mo

24 mo

Question . 23. The probable age of a child who rolls back to front, has a thumb-finger grasp,
self-inhibits to "no," and bangs two cubes is:

7-8 mo
Explanation: These milestones correspond respectively to control of
exploration, exploration of small objects, response to tone, and
comparison of objects. (See Chapter 10 in Nelson Textbook of Pediatrics,
17th ed.)

10-12 mo

12-15 mo

3-4 mo

15-18 mo
Question . 24. Growth between 3 and 4 mo of age is best characterized as:

Accelerating to a rate of 45 g/day

Slowing to a rate of 10 g/day

Slowing to a rate of 20 g/day


Explanation: This is a normal response that does not cause the child to
cross growth percentiles in a growth chart. (See Chapter 10 in Nelson
Textbook of Pediatrics, 17th ed.)

Accelerating to a rate of 20 g/day

Demonstrating no change compared with rate between 0 and 2 mo

Question . 25. Feeding between 6 and 12 mo of age is characterized by all of the following
except:

Being willing to be fed by a stranger


Explanation: Indeed, stranger anxiety may start to set in at this time. (See
Chapter 10 in Nelson Textbook of Pediatrics, 17th ed.)

Appearing autonomous

Eating finger foods

Turning away from the spoon

Holding a spoon

Question . 26. The probable age of a child who skips, names four colors, and dresses and
undresses is:

15 mo

24 mo

30 mo

18 mo

60 mo
Explanation: This is normal, appropriate motor, language, and social
development. (See Chapter 11 in Nelson Textbook of Pediatrics, 17th ed.)
Question . 27. Early walking suggests:

Preoccupation with objects

Advanced social development

Advanced language development

High-activity type
Explanation: More active types tend to walk early. The other choices are
incorrect. (See Chapter 11 in Nelson Textbook of Pediatrics, 17th ed.)

Spasticity

Question . 28. Between 2 and 5 yr of age, language increases; as a rule, the number of words in
a sentence is:

Based on knowledge of numbers

Equal to the age of the child in years


Explanation: A 2-yr-old has two-word sentences, such that language is
dependent on the environment and verbal interactions with adults. (See
Chapter 12 in Nelson Textbook of Pediatrics, 17th ed.)

Independent of the environment

Independent of the number of questions asked the child by adults

Based on the ABCs

Question . 29. All of the following statements regarding language development are true except:

Deaf children may create their own language

The basics for language may be "hard-wired" in the brain

Language has no role in behavior regulation


Explanation: Language plays a critical part in the regulation of behavior
as children internalize speech. (See Chapter 12 in Nelson Textbook of
Pediatrics, 17th ed.)

Delayed language may signify deafness

Delayed language may signify mental retardation


Question . 30. Growth during the years between 6 and 12 yr is characterized by annual weight
and height increments of:

3.5 kg, 6 cm
Explanation: 3.5 kg (7 lb) and 6 cm (2.5 in) per year are the average
increments in this time period. (See Chapter 13 in Nelson Textbook of
Pediatrics, 17th ed.)

6 kg, 3.5 cm

5 kg, 10 cm

10 kg, 5 cm

1.5 kg, 5 cm

Question . 31. Word-finding difficulties may result in all of the following except:

Difficulty in expressing feelings

Difficulty in verbal self-defense

Frustration

Success in English class


Explanation: Language-based classes produce difficulties for children
with word-finding difficulties. (See Chapter 13 in Nelson Textbook of
Pediatrics, 17th ed.)

Physical acting out

t
Psychologic Disorders, Set 1
Question . 1. Neuroleptic antipsychotic agents produce all of the following unwanted side
effects except:

Bradykinesia

Hyperthermia

Tardive dyskinesia

Inappropriate secretion of ADH


Explanation: Common complications of neuroleptic antipsychotic agents
include extrapyramidal symptoms (Parkinson-like syndrome), sedation,
and anticholinergic symptoms. Neuroleptic malignant syndrome
(malignant hyperthermia) and tardive dyskinesia are rarer complications.
(See Chapter 27 in Nelson Textbook of Pediatrics, 17th edition.)

Sedation

Question . 2. A 4-yr-old boy is noted to have stereotypic body movements and poor verbal and
nonverbal communication, with absence of empathy. At daycare, he has not made any friends.
The most likely diagnosis is:

Attention deficit hyperactivity disorder

Dysthymic syndrome

Deaf-mutism

Autism
Explanation: Autism is a disease of unknown cause and is more common
in males. It is characterized by the symptoms noted in this patient, with
onset usually before 30 mo of age. (See Chapter 27 in Nelson Textbook
of Pediatrics, 17th edition.)

Cerebral palsy

Question . 3. Head banging, hair twirling, rocking, thumb sucking, teeth grinding, and nail biting
all are:

Habit disorders that probably relieve tension


Explanation: The belief that choice B is correct leads to repeated
attempts to "break" children (a distressing term and concept), probably
in effect reinforcing the behavior. (See Chapter 21 in Nelson Textbook of
Pediatrics, 17th edition.)

Behavior problems that are easy to cure in children

Evidence of insecurity in the majority of children and poor parenting by


their parents

Tics
Question . 4. Which of the following statements about Gilles de la Tourette syndrome is true?

It is characterized by tics and coprolalia


Explanation: Gilles de la Tourette syndrome, which has a lifetime
prevalence rate of 0.5 per 1000 persons, is a rare condition in children. It
is characterized by multiple tics, compulsive barking, and shouting
obscene words (coprolalia). It is more common in first-degree relatives of
patients with Tourette syndrome than in the general population and
affects boys three to four times more frequently than girls. The cause is
uncertain, but research has shown that drugs that increase
dopaminergic action precipitate or worsen both tics and Tourette
syndrome. Many environmental precipitants have been noted to serve as
emotional stress sources. The syndrome can be fairly well managed with
haloperidol, a dopaminergic antagonist. Anecdotal reports in the
literature suggest that the serotonin reuptake inhibitors are also
efficacious in its treatment. (See Chapter 21 in Nelson Textbook of
Pediatrics, 17th edition.)

It is characterized by tics and encopresis

It is treated with haloperidol and methylphenidate

It is a common disorder of childhood

It affects girls more often than boys

Question . 5. Night terrors are associated with:

REM sleep

Overeating after 7:00 p.m.

The use of antipsychotic medication

Inception in preschool years and occasional somnambulism


Explanation: Night terrors most commonly occur during stage IV, deep
sleep. Neither the use of antipsychotic medications nor overeating after
7:00 p.m. has ever been shown to be associated with night terrors. They
usually begin in the preschool years. A child having night terrors is
confused and disorientated and shows signs of intense autonomic
activity (labored breathing, dilated pupils, sweating, tachypnea,
tachycardia). Sleepwalking may occur during night terrors and may put a
child at risk for injury. The incidence of night terrors is said to be
between 1% and 4% and is greater in boys. There is a familial pattern in
the development of the symptoms, and febrile illnesses may serve as
precipitating factors. (See Chapter 20.5 in Nelson Textbook of Pediatrics,
17th edition.)

Anger within the family


Question . 6. Conduct disorder in childhood and adolescence is associated with all of the
following except:

Antisocial behavior

Criminality in the father

Physical abuse

Marital discord within the home

Mental retardation
Explanation: Conduct disorder is a distinct clinical entity manifested by
several different antisocial behaviors: stealing, lying, fire setting,
truancy, property destruction, cruelty to animals, rape, use of weapons
while fighting, armed robbery, physical cruelty to others, and repeated
attempts to run away from home. Many argue that conduct disorder is
not a unitary illness but instead comprises three different syndromes
characterized primarily by aggression, intermittent antisocial behaviors,
and delinquency. Little is known about the antecedents of each of these
subtypes or the outcome of patients suffering from them. Risk factors
associated with the development of conduct disorders include antisocial
behavior within family members, criminality in the father, physical abuse
within the home, and marital discord within the home. Many different
approaches have been used in the treatment of children and adolescents
with aggressive behavior, antisocial behavior, and delinquency. The
most effective results have been obtained with parent training
management, in which parents are trained directly to promote prosocial
behaviors within the home and to place reasonable limits on unwanted,
destructive behavior. (See Chapter 25 in Nelson Textbook of Pediatrics,
17th edition.)

Question . 7. Completed suicides in childhood and adolescence may be associated with all of
the following except:

Previous suicide attempts

Alcohol or drug abuse

A history of depression and suicide within the family

Easy access to firearms

Perfectionism within the classroom


Explanation: Previous suicide attempts, substance abuse, easy access
to firearms, and a family history of depression and suicide are all closely
related to both suicide attempts and completed suicides in both children
and adolescents. Fifteen to 40% of completed suicides are preceded by
other suicide attempts. In more than one third of suicides, a parent, a
sibling, or another first-degree relative has previously shown overt
suicidal behavior. Firearms serve as the major method of death in
adolescent suicide. Among preadolescents, jumping from heights is the
most common method. Although perfectionism in the classroom has
been shown to be associated with specific types of anxiety, no
correlation has been shown between perfectionism and suicidal ideation
or suicidal behavior. (See Chapter 24 in Nelson Textbook of Pediatrics,
17th edition.)

Question . 8. A child in the third grade has problems with spelling and reading. She appears
very quiet and confused in class. Her teacher has noticed that this girl has trouble following
directions. Her mind seems to wander whenever the teacher tells a story or explains something
complicated. She is skilled in art and so far has performed well in arithmetic. Which of the
following diagnostic procedures is most likely to yield useful findings in this child?

An attention deficit questionnaire

An intelligence test

A language evaluation
Explanation: The question offers a classic example of a language
problem, particularly in a child with partial understanding. Many children
or parents do not admit to this problem. (See Chapter 29 in Nelson
Textbook of Pediatrics, 17th edition.)

A psychiatric assessment to rule out depression

A neurologic examination

Question . 9. An 11-yr-old child has excellent ideas in a class discussion, but what she records
on paper is primitive and unsophisticated. She can spell well in isolation and understands rules
of punctuation and capitalization, but in her own writing she makes multiple errors and mistakes
in punctuation and capitalization. Her handwriting is legible, but writing is painfully slow. This
girl most likely is having problems with:

Expressive language

Graphomotor production

Ideation

Attention

Simultaneous retrieval memory


Explanation: Simultaneous retrieval memory defects are depicted by the
history of the child described in the question. (See Chapter 29 in Nelson
Textbook of Pediatrics, 17th edition.)
Question . 10. Somatoform disorders, as part of psychosomatic illness, include all of the
following except:

Conversion reaction

Asthma
Explanation: Asthma may be exacerbated by psychologic factors and is
therefore a psychophysiologic disorder. (See Chapter 19 in Nelson
Textbook of Pediatrics, 17th edition.)

Hypochondriasis

Pain disorders

Question . 11. Mnchausen syndrome by proxy is characterized by all of the following except:

Recurrent illness that cannot be explained

Statements from experienced pediatricians that they have never seen


such a case

Symptoms that disappear with the parent present


Explanation: In the Mnchausen by proxy syndrome, a patient's
symptoms disappear if the parent goes home. Manifestations reappear
when the involved parent returns to the hospital. (See Chapter 35 in
Nelson Textbook of Pediatrics, 17th edition.)

An attentive parent caregiver who never goes home

An unworried parent caregiver

Poor response to therapy

Doctor shopping

Question . 12. When a 7-yr-old child fails to cooperate with care in the hospital, one should
suspect:

Immaturity

Embarrassment

Negativism

Fearfulness
Explanation: Children may be frightened and react to fear with a personal
manner of withdrawal or poor cooperation. (See Chapter 17 in Nelson
Textbook of Pediatrics, 17th edition.)

Oppositionism
Question . 13. If a parent does not appear readily reassured by the diagnosis or treatment plan,
one should suspect:

Hidden anxiety
Explanation: Parents often hold back questions that are highly charged
or that may appear "stupid." They may be angry, ashamed, or
uncomfortable in asking these questions. (See Chapter 17 in Nelson
Textbook of Pediatrics, 17th edition.)

Mistrust

Negativism

Oppositionism

Aggression

Question . 15. Psychosocial problems may manifest as disturbances in:

Feelings

Body function

Behavior

Performance

All of the above


Explanation: Disturbances may manifest as depression, anxiety,
aggression, and learning problems. (See Chapter 27 in Nelson Textbook
of Pediatrics, 17th edition.)

Question . 16. Psychiatric disorders are more common than in the general population of children
in all of the following except:

Smart students
Explanation: An important aspect of these conditions is the capacity of
the parents to adjust and cope. (See Chapter 18 in Nelson Textbook of
Pediatrics, 17th edition.)

Head trauma

Mental retardation

Epilepsy

Prematurity

Encephalitis
Question . 17. Conversion reactions are best characterized by:

Sudden onset

Traceability to a precipitating event

Involvement of special senses

Pseudoseizures

Abrupt end

All of the above


Explanation: The loss or alteration of function without a demonstrable
organic cause defines a conversion reaction, a type of somatoform
disorder. (See Chapter 19 in Nelson Textbook of Pediatrics, 17th edition.)

Question . 18. Enuresis is defined as:

Wetting 2 times per week for 3 consecutive mo


Explanation: Alternatively, it may be defined as wetting that produces
clinically significant distress for the child. (See Chapter 20 in Nelson
Textbook of Pediatrics, 17th edition.)

Wetting 2 times per week for any 3 mo in a year

Not being dry at 3 yr of age

Not being dry at 4 yr of age

Wetting at 5 yr of age on two occasions

Question . 19. Nocturnal enuresis is described by all of the following except:

Being primary or secondary

Having a strong genetic component

Occurring at all stages of sleep

When primary, being associated with emotional disorders


Explanation: There is no higher rate of emotional disturbances in age-
matched enuretic and nonenuretic children. (See Chapter 20 in Nelson
Textbook of Pediatrics, 17th edition.)

Being more common in males


Question . 20. Treatment of enuresis should include all of the following except:

Enlisting the cooperation of the child

Having the child void before retiring

Using alarms

Having the child launder the soiled sheets

Waking the child repeatedly


Explanation: Repeated waking may be beneficial for a few children but
creates added stress and hostility. (See Chapter 20 in Nelson Textbook
of Pediatrics, 17th edition.)

Question . 21. A 5-yr-old is noted by the parents to snore at night. The child has also had
problems staying awake in preschool and has had behavioral problems. The father also snores.
Physical examination of the child reveals large, pink, nonexudative tonsils. The most
appropriate next step is:

Laryngoscopy

Polysomnography
Explanation: Polysomnography reveals episodes of apnea and hypoxia.
(See Chapter 20 in Nelson Textbook of Pediatrics, 17th edition.)

Ambulatory apnea monitoring

Telemetry

Arterial blood gas analysis

Question . 22. The most likely diagnosis of the patient in Question 21 is:

Tonsillitis

Peritonsillar abscess

Obstructive sleep apnea syndrome


Explanation: Obstructive sleep apnea syndrome (OSAS) commonly
presents with airway obstruction during sleep with resultant sleep
disturbances at night and daytime sleepiness. (See Chapter 20 in Nelson
Textbook of Pediatrics, 17th edition.)

Tangier disease

Narcolepsy
Question . 23. The appropriate therapy for severe obstructive sleep apnea syndrome is:

Adenotonsillectomy
Explanation: Removal of the hypertrophied tissue relieves the symptoms
of OSAS. (See Chapter 20 in Nelson Textbook of Pediatrics, 17th edition.)

Tracheostomy

Parapharyngeal muscle surgery

Theophylline

Bilevel positive airway pressure

Question . 24. Risk factors for obstructive sleep apnea syndrome include all of the following
except:

Retroposition of the mandible

Small triangular chin

Long oval face

Long or soft palate

All of the above


Explanation: All of the answers are risk factors as well as the more
obvious such as Pierre Robin syndrome and Prader-Willi syndrome. (See
Chapter 20 in Nelson Textbook of Pediatrics, 17th edition.)

Question . 25. All of the following are considered habit disorders except:

Tics

Bruxism

Trichotillomania

Stuttering
Explanation: Stuttering is often discussed with habit disorders; however,
it is probably not a true habit in that it is not regarded as a tension-
relieving activity. (See Chapter 21 in Nelson Textbook of Pediatrics, 17th
edition.)

Thumb sucking
Question . 26. Tics are characterized by all of the following except:

Difficulty in controlling behavior

Occurrence of brief, transient amnesia after the tic


Explanation: This would be more compatible with a minor seizure. (See
Chapter 21 in Nelson Textbook of Pediatrics, 17th edition.)

Disappearance during sleep

Possible occurrence following encephalitis

Normal EEG

Question . 27. All of the following are true of Tourette syndrome except:

It is made worse by drugs that increase dopaminergic action

Some patients have pediatric autoimmune neuropsychiatric disorder

Some patients have oppositional defiant disorder

Lyme disease may sometimes mimic Tourette syndrome

Management with haloperidol or pimozide is often unsuccessful


Explanation: Both drugs provide a fair degree of relief. (See Chapter 21 in
Nelson Textbook of Pediatrics, 17th edition.)

Question . 28. A third-grade student refuses to go back to school after the winter break. She now
needs her mother to go to sleep with her and complains of headache, bellyache, and muscle
pain. Findings on physical examination are totally normal, but you notice the child is very clingy
to the mother. The most likely diagnosis is:

Stranger anxiety

School anxiety

Stranger reaction

Separation anxiety disorder


Explanation: Parents often consciously or unconsciously encourage this
intense fear of separation because of a fear that something will happen
to the caregiver. (See Chapter 22 in Nelson Textbook of Pediatrics, 17th
edition.)

Narcolepsy
Question . 29. Obsessive-compulsive disorder may be associated with all of the following
except:

Overconcern with body wastes

Prior group A streptococcal infection

Excessive fears

A need for sameness

Increased metabolic activity in the corpus callosum


Explanation: Positron emission tomography may reveal excessive
metabolic activity in the frontal lobes and basal ganglia. (See Chapter 22
in Nelson Textbook of Pediatrics, 17th edition.)

Excessive checking of locks

Question . 30. Major depression is characterized by:

Weight loss

Weight gain

Insomnia

Hypersomnia

Dysphoria

All of the above


Explanation: Major depression may also include manifestations of
fatigue, loss of interest or pleasure (dysphoria), agitation or retardation,
excessive guilt, feelings of worthlessness, and poor concentration. (See
Chapter 23 in Nelson Textbook of Pediatrics, 17th edition.)

Question . 31. Major depression in children may be characterized by:

A strong genetic component


Explanation: Twin studies show a strong genetic component to major
depression. (See Chapter 23 in Nelson Textbook of Pediatrics, 17th
edition.)

Delay of onset until after adolescence

Absence of hallucinations

No risk of depression in adulthood

None of the above


Question . 32. The treatment of choice for childhood-onset major depression is:

Monoamine oxidase inhibitors

Tricyclic antidepressants

Serotonin reuptake inhibitors


Explanation: SSRIs have proven efficacy and have a reasonable safety
profile. (See Chapter 23 in Nelson Textbook of Pediatrics, 17th edition.)

Benzodiazepines

None of the above

Question . 33. All of the following statements about suicide are true except:

15-40% of completed suicides are preceded by attempts

There are 4-5 attempts for each suicide

Access to guns increases the risk of suicide

Alcohol use is unrelated to suicide


Explanation: Use of alcohol and other drugs is associated
epidemiologically with suicide. (See Chapter 24 in Nelson Textbook of
Pediatrics, 17th edition.)

Depression is related to suicide

Question . 34. Important questions after an attempted suicide include all of the following except:

Is the patient less depressed?

Is the patient physiologically stable?

Does the patient still want to die?

Are precipitating events still active?

Does the patient have a future view or orientation?

Have the shame and guilt been moderated?

All of the above


Explanation: These are all helpful in discharge planning and in assessing
recurrence risks. (See Chapter 24 in Nelson Textbook of Pediatrics, 17th
edition.)
Question . 35. A 2-yr-old doesn't get his way in a crowded toy store. He starts to cry and hit and
roll on the floor. The parent should do all of the following except:

Yell and punish him


Explanation: Caregiver anger reinforces the child's behavior and teaches
a vicious cycle of oppositional behavior. (See Chapter 25 in Nelson
Textbook of Pediatrics, 17th edition.)

Acknowledge the child's frustration

Quietly explain that his response is not acceptable

Give him time and space to recover

Nonemotionally place him on a time out

Question . 36. A 4-yr-old is seen hitting his sister. When asked what he has done, he lies. His
lying is most likely to represent:

Pathologic behavior, which needs punishment

Displacement

Reaction formation

Avoidance of an unpleasant punishment

None of the above


Explanation: Lying is common at every developmental age and has
reasons at each level of development. It may include fantasy, avoidance
of punishment, and fear. Pathologic repeated chronic lying often occurs
in combination with other antisocial behavior. At any time, lying should
be approached calmly with an understanding of the fears of the child.
Nonetheless, honesty must be reinforced. (See Chapter 25 in Nelson
Textbook of Pediatrics, 17th edition.)

Question . 37. All of the following statements regarding minor stealing are true except:

All (almost) children steal at least once

It must be overemphasized to avoid a second episode


Explanation: Overemphasis may create an excitement to repeat the
offense. (See Chapter 25 in Nelson Textbook of Pediatrics, 17th edition.)

They should return the stolen item

It may be learned from parents

It may be impulsive
Question . 38. A 26-mo-old boy has a history of poor speech development, tantrum-like rages,
and rocking, repetitive, ritualistic behavior. He attends daycare but spends most of his time in
solitary play. The most likely diagnosis is:

Encephalitis

Latent slow virus infection

Rasmussen disease

Autism
Explanation: Autism begins before 30 mo of age and has a 4:1 male-to-
female ratio. It is characterized by impaired verbal and nonverbal
communication, imaginative activity, and reciprocal social interaction.
(See Chapter 27 in Nelson Textbook of Pediatrics, 17th edition.)

Prader-Willi syndrome

Question . 39. Additional features of the disease affecting the patient described in Question 38
include all of the following except:

Empathy
Explanation: Lack of social relations and absent empathy are typical of
children with autism. (See Chapter 27 in Nelson Textbook of Pediatrics,
17th edition.)

Preoccupation with body parts

Prevalence of 3/1000-4/1000 population

Association with fragile X

Visual scanning of the fingers

Question . 40. All of the following statements regarding attention deficit hyperactivity disorder
are true except:

It is more common in males than in females

Tic disorders may coexist with ADHD

Patients dislike or avoid sustained mental efforts

Oppositional defiant disorders never coexist with ADHD


Explanation: Co-morbidity includes oppositional defiant disorder in 50%,
conduct disorders in 30-50%, anxiety disorder in 20-25%, mood disorders
in 15-20%, and learning disorders in 10-25%. (See Chapter 29 in Nelson
Textbook of Pediatrics, 17th edition.)

Substance abuse occurs in affected adolescents


Question . 41. Possible complications of stimulant drug therapy for attention deficit
hyperactivity disorder include all of the following except:

Jitteriness

Difficulty sleeping

Abdominal pain

Tics

Increased appetite
Explanation: Anorexia is common; this affects growth. (See Chapter 29 in
Nelson Textbook of Pediatrics, 17th edition.)

Question . 42. Which of the following statements is false?

The clinical interview is a tool for gathering information, not for


enhancing behavior
Explanation: (See Chapter 17 in Nelson Textbook of Pediatrics, 17th
edition.)

Most adolescents do not experience more stress during this


developmental stage than at others

Some "symptomatic" actions of children may be part of normal


development

Chronic stresses are more difficult for a child to manage than a single
acute stressful episode

Infants tend to react to stressful situations with impaired physiologic


functions

Question . 43. Which of the following statements is false?

Head injury is relatively common in pediatric populations

Serious brain injuries early in life may be more generally incapacitating


than those that occur later

There is an increased risk of a psychiatric disorder after brain injury

Psychosis is often seen as a result of brain injury in childhood


Explanation: (See Chapter 18 in Nelson Textbook of Pediatrics, 17th
edition.)

Even when head injury is judged to be mild, there may be educational


problems in 25-50% of affected children
Question . 44. Which of the following statements is false?

In somatoform disorders symptoms are associated with unconscious


conflict, but in factitious disorders the unconscious need to be cared for
motivates the falsification of symptoms

Conversion disorder is a type of somatoform disorder that usually


presents in early childhood
Explanation: (See Chapter 19 in Nelson Textbook of Pediatrics, 17th
edition.)

Pseudoseizures are the most common conversion symptom

10-15% of children have ongoing somatic symptoms

Psychoactive medications may be useful in treating anxiety or


depression in childhood

Question . 45. Which of the following choices is not a helpful treatment for enuresis?

Desmopressin acetate

Imipramine

Maintaining a star chart

Bell-and-pad apparatus

Punishment
Explanation: (See Chapter 20 in Nelson Textbook of Pediatrics, 17th
edition.)

Question . 46. Each of the following disorders is often co-morbid with Tourette syndrome
except:

Borderline personality disorder


Explanation: (See Chapter 21 in Nelson Textbook of Pediatrics, 17th
edition.)

Obsessive-compulsive disorder

Attention deficit hyperactivity disorder

Oppositional defiant disorder

Tic disorder
Question . 47. A 7-yr-old boy is referred to you by the school for problems with hyperactivity,
disruptive behavior, and stomach aches. They report that he "acts up" in class and have
requested that the parents pick him up from school early on several occasions. The child
presents to your clinic with his mother. He does not appear fidgety during the evaluation. His
mother reports few problems at home except that he complains of stomach aches when he
wakes up in the morning. His mother appears depressed and "tears up" while sharing that her
husband was recently diagnosed with cancer. You diagnose the child to be suffering from
separation anxiety disorder and recommend the following:

A trial of an SSRI for the child

A trial of dexedrine or methylphenidate

Tell the mother she needs to be strong for the rest of the family

Referral of the family for therapy


Explanation: (See Chapter 22 in Nelson Textbook of Pediatrics, 17th
edition.)

Obtain permission from school officials for the child to be home-


schooled for the rest of the semester

Question . 48. A 12-yr-old girl presents with a history of repetitive handwashing related to
contamination obsessions. After performing an interview you diagnose OCD and begin
treatment with sertraline. After an initial response, you increase the dose to the recommended
maximum dose. She tolerates the medication without side effects but continues to be impaired
by the obsessions and washing rituals. The next step in treatment involves:

Increasing the sertraline

Adding fluvoxamine to the sertraline

Referring the girl for cognitive-behavioral therapy


Explanation: (See Chapter 22 in Nelson Textbook of Pediatrics, 17th
edition.)

Adding risperidone to the sertraline

Obtaining an ASO titer

Question . 49. A 9-yr-old boy who witnessed the murder of his mother by his father is referred to
you by the school psychologist. She has treated the child for 9 mo and is concerned that the
child is quite "jumpy" at school. In addition, she states that his grades have fallen because he
cannot pay attention in class. He reports difficulty falling asleep at night. Which of the following
statements concerning the use of medications for the treatment of post-traumatic stress
disorder (PTSD) in children is true?

Clonidine and guanfacine are helpful for sleep disturbance and


persistent arousal
Explanation: (See Chapter 22 in Nelson Textbook of Pediatrics, 17th
edition.)
Stimulants are the treatment of choice for inattention in PTSD

SSRI-medications are not helpful in the treatment of co-morbid


depression, anxiety, and withdrawal.

Alprazolam and lorazepam are recommended treatments for anxiety and


sleep problems associated with PTSD

Too much time has passed after the trauma, so medications will be of
little help

Question . 50. The lifetime prevalence for depression starting in adolescence is:

2-4%

6-8%

15-20%
Explanation: (See Chapter 23 in Nelson Textbook of Pediatrics, 17th
edition.)

35-50%

60-70%

Question . 51. All of the following statements about adolescent suicide are true except:

It is the leading cause of death in adolescence

Mood problems and violence are major risk factors

Substance abuse is a major risk factor

The suicide rate is higher in younger adolescents than in older


adolescents
Explanation: (See Chapter 24 in Nelson Textbook of Pediatrics, 17th
edition.)

Previous suicide attempts are a major risk factor

Question . 52. Which of the following statements is false?

Breath holding is not unusual during the first year of life

In response to tantrums, parents should be advised to give the child time


and space to recover by turning away briefly
Lying is more common in children with high self-esteem
Explanation: (See Chapter 25 in Nelson Textbook of Pediatrics, 17th
edition.)

Truancy and run-away behavior are never developmentally appropriate

Children exposed to aggressive models on television display more


aggressive behavior than that noted in children not so exposed

Question . 53. Which of the following statements about homosexuality in adolescence is true?

The prevalence is less than 1%

It is a risk factor for suicide, especially in males


Explanation: (See Chapter 26 in Nelson Textbook of Pediatrics, 17th
edition.)

It is designated as a psychologic disorder by the American Psychiatric


Association

Behavioral treatments for homosexuality have been shown to be


effective in changing orientation

It is associated with various psychologic disorders in adulthood

Question . 54. Which of the following statements is false?

Autism is typically diagnosed after 36 mo


Explanation: (See Chapter 27 in Nelson Textbook of Pediatrics, 17th
edition.)

Ritualistic behavior is common with autism

In autism, intelligence measured by conventional psychological testing


usually falls in the functionally retarded range

Genetic factors play a significant role in autism

Early, intensive behavioral therapy, targeted toward speech and


language development, is successful in improving language and social
function of autistic children
Question . 55. Which of the following medications is not used as a mood stabilizer?

Depakote

Lithium carbonate

Carbamazepine

Topiramate

Fluoxetine
Explanation: (See Chapter 28 in Nelson Textbook of Pediatrics, 17th
edition.)

Question . 56. All of the following statements are true except:

Chromosome abnormalities can lead to unique patterns of language


defects

Children who can register and consolidate facts and procedures in


memory may have great difficulty accessing or retrieving these items
when they are needed

Children with special ordering weaknesses do not have problems with


letter and word recognition
Explanation: (See Chapter 29.1 in Nelson Textbook of Pediatrics, 17th
edition.)

Children with special ordering weaknesses may be late in discriminating


between right and left

Dyspraxia in general relates to difficulty in developing an ideomotor plan


and activating coordinated motor actions to complete a task

Question . 57. All of the following statements are true except:

ADHD frequently occurs with other emotional, behavioral, language, and


learning disorders

ADHD is 3 to 4 times more common in males than in females

Several standard behavior rating scales do well in discriminating


between children with ADHD and controls, and these scales are
sufficient to make the diagnosis
Explanation: (See Chapter 29.2 in Nelson Textbook of Pediatrics, 17th
edition.)
Depression and anxiety disorder may present many of the same
symptoms as those of ADHD

Psychosocial interventions, behavior management training, and


medication are effective in treating various components of ADHD

Question . 58. A 20-yr-old college student is referred from the university health service because
of very poor grades during the previous semester. His class participation is good, he seems to
understand the concepts, but he is unable to finish his tests. His admission folder indicates that
he was diagnosed as having a reading problem in fourth grade and received special help in
grades 5-8. He was allowed to take his SATs untimed. Of the following, the most therapeutic
recommendation is:

Intensive phonologic awareness training

Provision of extra time on tests


Explanation: (See Chapter 29.3 in Nelson Textbook of Pediatrics, 17th
edition.)

Systematic, explicit instruction in phonics

Multisensory training

Referral to ophthalmology

Question . 59. An 8-yr-old girl is referred for evaluation of learning problems in school. Her
mother reports that her father, an independent plumbing contractor, had similar problems. The
child's motor milestones were all normal, but she did not say her first words until 18 months.
The most important element in making a diagnosis of dyslexia in this child is:

Verbal IQ tested at a standard score of 94

Poor balance on tandem gait testing

Many errors on a continuous performance test

History of aggressive behavior with classmates

Word reading test at a standard score of 84


Explanation: (See Chapter 29.3 in Nelson Textbook of Pediatrics, 17th
edition.)
Question . 60. Parents of a 12-yr-old girl are concerned because at a parent-teacher conference
they were told that their daughter does not seem to be keeping up with her schoolwork in
language arts, although her abilities in mathematics place her at the top of the class. She had
difficulty in early grades with reading but then seemed to catch up. Currently, she seems to be
able to read words accurately but reads very slowly. She reads the necessary school
assignments but does not read for pleasure. Which of the following is the most likely
explanation for this history?

Poor motivation

Inability to decode words

Low intelligence

Poor reading influence


Explanation: (See Chapter 29.3 in Nelson Textbook of Pediatrics, 17th
edition.)

Parent-child problems in the home


Social Issues

Question . 1. A 6-mo-old boy is brought to the emergency room and is afebrile but responds poorly to
tactile and auditory stimuli. He becomes apneic and unresponsive after a generalized seizure. The
parents state that he was perfectly well in the car on the way to the hospital and that they only brought
him to the emergency room because of constipation. He requires 10 min of cardiopulmonary
resuscitation, after which he is noticed to have a bulging fontanel and bilateral retinal hemorrhages. A
chest film reveals two posterior rib fractures. The most likely diagnosis is:

CPR-induced retinal hemorrhages and rib fractures

Hemorrhagic shock and encephalopathy

Hemophilia

Status epilepticus

Child abuse-shaken baby syndrome


Explanation: In the classic shaken baby syndrome, a computed tomography
scan of the head reveals diffuse cerebral edema and hemorrhage. CPR in
young children does not usually produce retinal hemorrhages and rarely, if
ever, produces rib fractures. (See Chapter 35 in Nelson Textbook of Pediatrics,
17th ed.)

Question . 2. A young child's response to the death of a parent often is characterized by:

Depression and weight loss

Denial and magical wishing


Explanation: Many young children continue in their daily activities and use
denial and magical wishful thoughts for reunion and reappearance. (See
Chapter 33 in Nelson Textbook of Pediatrics, 17th ed.)

Anger and crying

Wishes of death for himself or herself

None of the above

Question . 3. The effect that statements such as "stop it or you'll give me a headache" have on young
children is to:

Teach a child to behave

Give children a pattern of headaches

Create guilt and unrealistic fault


Explanation: Statements such as "stop it or you'll give me a headache" may
cause a child to suffer significant and unrealistic guilt, especially if the parent
leaves for some time or is hospitalized. (See Chapter 33 in Nelson Textbook of
Pediatrics, 17th ed.)
Provide parents with a way to cope

Prepare children for separation

Question . 4. All of the following statements about adoption are true except:

42% are stepparent or relative adoptions

Most adopted children are from foreign countries


Explanation: Five percent of adoptions are from overseas. (See Chapter 30 in
Nelson Textbook of Pediatrics, 17th ed.)

1 million children are adopted in the United States each year

15% are adopted through foster care

2-4% of American families have adopted children

Question . 5. All of the following statements regarding foster care are true except:

Chronic medical illness is present in 35% of children

60% of preschool children in foster care experience developmental delay

42% are white children

A majority of foster children receive EPSDT services


Explanation: Early prevention screening and developmental testing programs
are underutilized in the foster care system. (See Chapter 31 in Nelson
Textbook of Pediatrics, 17th ed.)

Children frequently stay at more than one foster care home

Question . 6. High-quality child care can influence all of the following except:

Child cognition

Future academic achievement

Social development

Sibling rivalry
Explanation: Sibling rivalry and aggression may not be affected as much as
the cognitive issues. (See Chapter 32 in Nelson Textbook of Pediatrics, 17th
ed.)

Scores on standardized tests


Question . 7. A 3-yr-old boy is missing from his mother's house approximately 1 mo after a divorce. The
most likely explanation is:

Sleepwalking

Drug reaction

Running away from his mother

Searching for his father


Explanation: It is not unusual for the young child to keep asking for the
missing parent, to wait at the door or window, or to go outside to look for the
parent. (See Chapter 33 in Nelson Textbook of Pediatrics, 17th ed.)

Anxiety reaction

Question . 8. After divorce, children may demonstrate all of the following except:

A feeling of being overburdened by residence in two homes

Withdrawal

Indifference at times of reunions

Academic deterioration

Expectations that the parents will never get back together


Explanation: Indeed, most children fantasize about the possibility that their
parents will remarry. (See Chapter 33 in Nelson Textbook of Pediatrics, 17th
ed.)

Question . 9. All of the following are age-related behavioral responses to experiencing violence except:

Infants-poor sleep

Adolescents-short-fuse responses

Toddlers-excessive appetite
Explanation: All the rest are true, as well as poor appetite, decreased
exploration of the environment in toddlers, and poor school performance with
hyperactivity in older children. (See Chapter 34 in Nelson Textbook of
Pediatrics, 17th ed.)

Toddlers-clingy behavior

School age-post-traumatic stress syndrome


Question . 10. Munchausen syndrome by proxy is characterized by all of the following except:

10% mortality

Multiple hospitalizations

Induced manifestations by caregiver

Ready admission of abuse by parents


Explanation: Parents often deny their involvement in inducing symptoms in
their children and will rapidly change doctors if it is discussed with them. (See
Chapter 35 in Nelson Textbook of Pediatrics, 17thed.)

Use of medications or toxins

Question . 11. Factors that may be associated with an increased risk of child abuse include:

Poverty

Military base residence

Spouse abuse

Unplanned pregnancy

All of the above


Explanation: Each is a risk factor. Of note, abuse has been reported in all
communities and from all socioeconomic levels. (See Chapter 35 in Nelson
Textbook of Pediatrics, 17th ed.)

Question . 12. A 2-mo-old is admitted with a fracture of the right femur. The mother states that the baby
fell off a low couch onto a plush carpeted floor and did not cry. Thereafter, the baby appeared fine.
Three days later, the grandmother noted that the baby cried when she changed the diaper and that the
leg was swollen. In the emergency department, a bruise was noted over the sternum that was also said
to have occurred during the fall 3 days ago. The mother states that she bleeds easily, but that the
father of the baby is well. An x-ray film reveals a spiral fracture of the child's femur. Features of this
case suggestive of abuse include all of the following except:

Multiple sites of injury

Implausible explanation for injury

Grandmother's deep concern


Explanation: Often, relatives do not know the nature or the cause of the injury,
nor do they readily know or admit to the identity of the person who committed
the trauma. (See Chapter 35 in Nelson Textbook of Pediatrics, 17th ed.)

Injury incompatible with the nature of the fall

Delay in seeking medical attention


Question . 13. Cardiopulmonary resuscitation in a child with head trauma from abuse commonly
results in all of the following except:

Recovery of a pulse

Retinal hemorrhages and broken ribs


Explanation: Retinal hemorrhages and broken ribs rarely follow CPR. The
nature of rib fractures is also different in abuse. (See Chapter 35 in Nelson
Textbook of Pediatrics, 17th ed.)

Recovery of respirations

Normal sinus rhythm

Recovery from cyanosis

Question . 14. Sexual abuse includes all of the following except:

Exposing sexual anatomy

Touching genitals by two preadolescents


Explanation: This is sexual play and is usually normal exploratory behavior if
there is no force or coercion and the children are not more than 4 years apart
in age. (See Chapter 35 in Nelson Textbook of Pediatrics, 17th ed.)

Showing pornography to a child

Use of a child to create pornography

Incest

Question . 15. Pedophiles are best described as:

Being female

Never having repeated experiences

Seeking opportunities to be in contact with children


Explanation: Pedophiles often seek out positions and opportunities to be
around children. Certain children may be particularly vulnerable, such as
those with mental and physical handicaps. (See Chapter 35 in Nelson
Textbook of Pediatrics, 17th ed.)

Preferring females

Being highly violent


Question . 16. A 4-yr-old girl is admitted to the hospital for her third evaluation for vaginal bleeding.
The mother noted bright red blood on the child's underwear. Previous examinations revealed a normal
4-yr-old girl, Tanner stage 1, with normal external genitalia. Pelvic ultrasound results were normal, as
was the serum estradiol level. The hemoglobin and platelet counts were normal, as were the bleeding
time and coagulation studies. Findings on pelvic examination conducted under anesthesia also were
normal. The next step in the examination is to:

Determine the blood type of the blood on the underwear


Explanation: By DNA typing, the blood turned out to be the mother's. The
mother has diabetes and employs home glucose monitoring and would
purposely put blood on her daughter's underwear. (See Chapter 35 in Nelson
Textbook of Pediatrics, 17th ed.)

Interrogate the father

Isolate the parents and child

Determine von Willebrand factor levels

Measure fibronectin in the vagina

Question . 17. The most likely diagnosis for the child described in Question 16 is:

Precocious puberty

Sexual abuse

Vaginitis

Coagulopathy

Munchausen syndrome by proxy


Explanation: Munchausen by proxy brings attention to the child and caregiver.
Often, the perpetrator has some medical background. (See Chapter 35 in
Nelson Textbook of Pediatrics, 17th ed.)

Question . 18. All of the following statements regarding adoption are true except:

Federal law requires that children in foster care who cannot be safely returned
to their families within a reasonable period of time be placed with adoptive
families

Agencies in the United States that arrange international adoptions have no


legal obligation to obtain accurate and complete health histories on children
whom families are considering adopting
th
Explanation: (See Chapter 30 in Nelson Textbook of Pediatrics, 17 ed.)

Families should be encouraged to speak freely and repeatedly about adoption


with the adopted child, beginning in toddler years and continuing through
adolescence
Most adopted children adjust well and lead healthy, productive lives

Many children in foster care waiting to be adopted have "special needs"

Question . 19. Which of the following statements regarding foster care is true?

A permanency plan must be made for a child in foster care no later than 12 mo
from the child's entry into care
Explanation: (See Chapter 31 in Nelson Textbook of Pediatrics, 17th ed.)

A minority of children in foster care have a history of abuse or neglect

An increasing proportion of children entering foster care are adolescents

Children in foster care have low utilization rates for all types of care

A minority of foster care children have behavioral and adjustment problems

Question . 20. All of the following statements regarding child care are true except:

Nearly one half of employed mothers with 3- and 4-yr-old children use center
care as their primary supplemental care

High-quality child care does not influence the cognitive and social
development of disadvantaged children
th
Explanation: (See Chapter 32 in Nelson Textbook of Pediatrics, 17 ed.)

Middle-class children are not protected from the effects of poor-quality child
care

Licensure of child-care providers signifies that minimal health, safety, and


sanitary practices are being followed

Accreditation of a child-care provider suggests that a program is of sufficient


quality to promote children's development

Question . 21. All of the following statements are true except:

Recurrent separations tend to accustom children to separations and make


them less wary and guarded about reestablishing the relationship with an
absent parent
Explanation: (See Chapter 33 in Nelson Textbook of Pediatrics, 17th ed.)

Most bereaved families remain socially connected and expect that life will
return to some new sense of normalcy

School-aged children think more concretely than younger children, recognize


the permanence of death, and begin to understand biologic processes of the
human body

No specific grief sign, symptom, or cluster of behaviors identifies the child or


family in need of help with bereavement after the death of a family member

Medication, as a first line of treatment, rarely proves useful in normal or


uncomplicated grief reactions

Question . 22. All of the following statements regarding children and violence are true except:

The source of first exposure to violence for children is often television


Explanation: (See Chapter 34 in Nelson Textbook of Pediatrics, 17th ed.)

The violence children experience and witness has a profound impact on health
and development

High levels of witnessing violence place children at risk for psychological,


social, academic, and physical problems

Occasional wife battering is estimated to occur in 16% of all families

The most ubiquitous source of exposure to violence for children in the United
States is television

Question . 23. The source of first exposure to violence for children is often:

Community violence

War violence

School violence

Domestic violence
Explanation: (See Chapter 34 in Nelson Textbook of Pediatrics, 17th ed.)

Question . 24. The violence children witness affects their development in all of the following ways
except:

It influences how they view the world and their place in it

Children become more capable in motor function from the constant threat of
violence
Explanation: (See Chapter 34 in Nelson Textbook of Pediatrics, 17th ed.)

Fear may thwart their exploration of the world, which is essential in learning in
childhood

Higher exposure to violence correlates with poorer performance in school,


symptoms of anxiety and depression, and lower self-esteem

Question . 25. A 6-mo-old child is brought to your office with the chief complaint of leg swelling and
decreased leg movement of 3 days' duration. The mother, a single parent, said that the child awoke 3
days ago with the swelling and guarding. She was unaware of any trauma to the child but said that her
2-yr-old "plays rough" with the 6-mo-old. She said that she delayed coming to see you because she
had no transportation. An x-ray film reveals a new (less than 7 days old) spiral fracture of the femur.
Which of the following should be your next action?

Refer the child to an orthopedist for casting

Perform a skeletal series, refer the child to an orthopedist for casting, examine
and do a skeletal survey of the sibling

Perform a skeletal series, refer the child to an orthopedist for casting, examine
and do a skeletal survey of the sibling, and report the spiral fracture to
appropriate authorities as suspected physical abuse after screening the
mother for risk factors and informing the mother of your plan
Explanation: (See Chapter 35 in Nelson Textbook of Pediatrics, 17th ed.)

Tell the mother you suspect physical abuse and interrogate her until you get a
history; when the mother tells you that the child fell from the crib to a carpeted
floor and sustained the injury, you conclude that the finding is in keeping with
the fracture, with no need to report suspected child abuse

Because this fracture is probably due to osteogenesis imperfecta, send the


child for genetics evaluation and connective tissue biopsy for a definitive
diagnosis

Question . 26. The parents of a 4-yr-old girl are in the process of a divorce. The child returned from a
visit with the father yesterday. The mother, who has primary custody of the child, examined the child's
genitalia because the child complained of dysuria. She found the genitalia to be red and open to the
"size of a dime." She asked the girl if the father had done anything to her and the child remained silent.
The child has been reluctant to visit the father. The mother brings the child to you because she is
concerned that the child has been abused. Which of the following constitutes the most appropriate
next step in management?

With the mother in the room for support, interview the child by asking if the
father has ever done anything to her genitalia. If the child answers in the
affirmative, make a report to appropriate authorities.

Report what the mother has told you to appropriate authorities.

Ask the father to come to your office so that you can interview him. It is likely
that the mother has prompted the child.

Speak to the child with the mother out of the room. Begin with general
questions. Determine knowledge of body parts and private parts. Ask if
anything has happened to various body parts. If the child answers in the
affirmative, ask what happened. Do a complete examination including
inspection of the anus and genitalia. If the child gives any history suggestive
of abuse or has findings suggestive of trauma to the anus or hymen that are
unexplained, make a report to the appropriate authorities.
Explanation: (See Chapter 35 in Nelson Textbook of Pediatrics, 17th ed.)

Question . 27. A 6-mo-old child has a 2-mo history of daily projectile vomiting. She has been
hospitalized on two occasions. Findings on laboratory studies including an upper GI series and on
swallowing studies have been normal. The mother had similar symptoms as a child. A 3-yr-old sister
has been treated for gastroesophageal reflux. You are the fourth physician that the mother, a nurse,
has contacted to diagnose and treat these persistent symptoms. Inspection of hospital records
indicates that no vomiting took place during either hospital admission. The most appropriate next step
in management is:

Conduct a complete and detailed work-up for cyclic vomiting with referrals for
consultation to gastroenterology, neurology, and nephrology. Persist in your
evaluation until a diagnosis is made.

Have the mother bring you a sample of the vomitus. Test it for the presence of
ipecac. Perform toxicology studies. If results are positive, report as suspected
Munchausen syndrome by proxy to proper authorities.
Explanation: (See Chapter 35 in Nelson Textbook of Pediatrics, 17th ed.)

Place the child on treatment for gastroesophageal reflux in light of the high
rate of false-negative results on laboratory testing for this condition. If the
condition persists despite treatment, hospitalize the child and obtain surgical
consultation.

Confront the mother with lack of medical findings for the condition. Indicate
that you suspect she is lying about the symptoms. Request a psychiatric
consultation for the mother. Report your suspicions to proper authorities with
a request that the child remain in the home under weekly supervision by a
public health nurse.

Consult an allergist and request a work-up for food allergy. Put the child on a
restrictive diet and observe for symptoms.
Children with Special Health Needs, Set 1

Question . 1. A mentally retarded child with microphthalmia, microcephaly, chorioretinitis, and


a history of a neonatal petechial rash is most likely to have:

A chromosomal syndrome

TORCH infection
Explanation: This pattern of abnormalities is most compatible with a
congenital TORCH (toxoplasmosis, other, rubella, cytomegalovirus,
herpes simplex) infection. In addition, intrauterine and postnatal growth
retardation may be evident. (See Chapter 37.2 in Nelson Textbook of
Pediatrics, 17th ed.)

Fetal alcohol syndrome

Galactosemia

Hyperammonemia

Question . 2. Frequent problems of children with common chronic illnesses include all of the
following except:

Unpredictability

Pain

Expense

Multiple providers

Failure to graduate high school


Explanation: Many children with common chronic diseases of
childhood (e.g., asthma, seizures, diabetes, arthritis, cystic fibrosis,
sickle cell anemia) attend high school and graduate. (See Chapter 37 in
Nelson Textbook of Pediatrics, 17th ed.)

Isolation

Psychologic or behavioral problems


Question . 3. An infant with multiple grotesque congenital anomalies dies on the third day of
life. Her mother has not had an opportunity to see her before death owing to postpartum
complications. When informed of the baby's death, she says she wants to see her. She cannot
be moved from where she is receiving intensive care. Which of the following is the most
appropriate response to her request?

Tell her that she is too sick to see the baby

Tell her that she will be able to see the baby later

Take the baby to her bedside


Explanation: Parents who are not able to see their deformed neonate
may greatly exaggerate the perceived severity of any anomaly and may
have excessive feelings of guilt. Most parents benefit from seeing their
child with anomalies and often identify aspects of beauty or normalcy
with the help of a nurse or physician. (See Chapter 37 in Nelson
Textbook of Pediatrics, 17th ed.)

Tell her she would not want to see the baby

Tell her that it is too late for her to see the baby

Question . 4. The parents of a 10-yr-old girl with mental retardation are seeking information on
what to expect for her future. The youngster is in a mainstreamed educational program, is just
beginning to master simple reading skills, and has one close friend. The difficulty of long-term
prognostication aside, possible life goals for this child include:

Holding a regular job

Getting married

Having children

All of the above


Explanation: This child demonstrates important educational and social
milestones that are partially predictive of future achievements with
respect to these life goals. (See Chapter 37.2 in Nelson Textbook of
Pediatrics, 17th ed.)

Question . 5. A preschooler with Down syndrome is seen for a routine health supervision visit.
A knowledgeable clinician will pay particular attention to screening for problems that are
known to occur with increased frequency in children with this condition. Which of the
following conditions is least likely to be found in this child?

Atlantoaxial instability

Neurogenic bladder
Explanation: Neurogenic bladder is not typically encountered in Down
syndrome, but Hirschsprung disease may occur. (See Chapter 37.2 in
Nelson Textbook of Pediatrics, 17th ed.)

Hypothyroidism

Conductive hearing loss


Question . 6. A 3-yr-old boy with a limited vocabulary is referred for formal psychometric
testing and is found to have an IQ of 60. Findings on physical examination are essentially
unremarkable except for mild hypotonia. Appropriate initial laboratory studies include all of
the following except:

Karyotype, including test for fragile X

Audiologic evaluation

Cranial CT scans
Explanation: Cranial computed tomography is not indicated until other
evaluations are completed unless a patient has macrocephaly,
microcephaly, abnormal neurologic findings, or significant
dysmorphology. (See Chapter 37.2 in Nelson Textbook of Pediatrics,
17th ed.)

Formal speech and language evaluation

Question . 7. The general approach to management of the child with psychosocial failure to
thrive includes all of the following except:

Keeping meal time brief

Offering solid foods before liquids

Forcing the child to eat


Explanation: Forced feeding exacerbates abnormal psychosocial
tension between a child and his or her caregiver. (See Chapter 36 in
Nelson Textbook of Pediatrics, 17th ed.)

Minimizing environmental distractions

Minimizing the intake of water and juice

Question . 8. From the following list, choose the most appropriate way(s) to assess growth in
premature infants to diagnose failure to thrive.
1. Use corrected age (subtract weeks premature) until age 1-2 yr
2. Determine whether two major growth percentiles are crossed
3. Add additional weight as if the child had been born at term
4. Determine the weight to length ratio
5. Do not use head circumference until age 24 mo

1 only

1 and 2
Explanation: Corrected age rather than chronologic age is one solution.
The "real" gestational age of a 24-wk premature infant who is now 20 wk
old is 44 wk (1 month corrected age may also be valuable). Determining
whether growth percentiles are crossed is the other solution. (See
Chapter 36 in Nelson Textbook of Pediatrics, 17th ed.)
3 and 4

3 and 5

1 and 5

Question . 14. Major causes of failure to thrive include all of the following except:

Formula feeding
Explanation: In infants who are fed formula in sufficient amounts,
malabsorption is not a major cause of failure to thrive. (See Chapter 36
in Nelson Textbook of Pediatrics, 17th ed.)

Failure to provide sufficient calories

Failure to ingest sufficient calories

Failure to retain sufficient calories

Malabsorption

Question . 15. The leading cause of failure to thrive in infants between 0 and 3 mo of age is:

TORCH infection

Psychosocial pathology
Explanation: All must be considered, but B is No. 1. (See Chapter 36 in
Nelson Textbook of Pediatrics, 17th ed.)

Gastrointestinal reflux

Cystic fibrosis

Inborn errors of metabolism

Question . 16. Snoring and mouth breathing as a cause of failure to thrive suggest:

Streptococcal pharyngitis

Mononucleosis

Obstructive sleep apnea


Explanation: Adenoid hypertrophy or poor oropharyngeal motility
(possibly related to cerebral palsy) can cause failure to thrive from
obstructive sleep apnea. Mononucleosis is rare in infants. (See Chapter
36 in Nelson Textbook of Pediatrics, 17th ed.)

Anterior meningocele

Cerebral palsy
Question . 17. An 8-mo-old presents with failure to thrive. The past medical history includes
severe thrush and Candida diaper rash and recurrent otitis media with perforation. On physical
examination the patient has generalized lymphadenopathy and hepatosplenomegaly; there is
also bilateral parotitis. The most likely diagnosis is:

Mononucleosis

Familial histiocytosis

X-linked combined immunodeficiency

AIDS
Explanation: Babies with AIDS usually present with failure to thrive and
recurrent infections. Lymphadenopathy, parotitis, and
hepatosplenomegaly are classic features of AIDS in infants. (See
Chapter 36 in Nelson Textbook of Pediatrics, 17th ed.)

Psychosocial failure to thrive

Question . 18. All of the following are true regarding children with chronic illness except:

Children with disabilities rarely survive to adulthood


Explanation: Indeed, most survive to be adults. (See Chapter 37 in
Nelson Textbook of Pediatrics, 17th ed.)

6-7% of children has some limitation of activity

1-2% of children meet the definition of severe disability

Of disabled children, 40% have learning and developmental disorders

Of disabled children, 35% have chronic physical conditions

Question . 19. Principles of care for children with chronic diseases include all of the following
except:

Early detection

Amelioration of functional consequences

Designation of children with asthma as "asthmatics"


Explanation: Labeling depersonalizes the child and marks the child for
life. A disease should not define a child. (See Chapter 37 in Nelson
Textbook of Pediatrics, 17th ed.)

Prevention of secondary psychosocial handicaps

Treatment in the context of the family


Question . 20. The Individuals with Disabilities Education Act includes all of the following
except:

Support of state programs providing early intervention

Providing cash assistance for children with disabilities


Explanation: Supplemental security income programs provide cash
support for people of all ages with disabilities. (See Chapter 37 in
Nelson Textbook of Pediatrics, 17th ed.)

Education in the least restrictive manner

Support of the state programs providing special education

Question . 21. A sixth-grade child with chronic arthritis typically views the cause of chronic
illness as due to:

Germ theory
Explanation: At this age, germ theory is an easily comprehended but
unfortunately not always accurate view of chronic illness. Nonetheless,
it does help with compliance in taking medicine. (See Chapter 37 in
Nelson Textbook of Pediatrics, 17th ed.)

Punishment for bad behavior

Physiologic mechanisms

Not following rules

Failure to take medicines

Question . 22. Mental retardation is best classified by:

A system designating the degree as mild, moderate, severe, profound

The designators imbecile, retarded, functional

IQ percentiles

Support needs (intermittent, limited, extensive, pervasive)


Explanation: This functional approach places the child's needs for
support into the context of the environment. (See Chapter 37 in Nelson
th
Textbook of Pediatrics, 17 ed.)

None of the above


Question . 23. Common identifiable causes of mental retardation include all of the following
except:

Trisomy 21

Hypothyroidism

Fetal alcohol syndrome

Fragile X syndrome

Cystic fibrosis
Explanation: Cystic fibrosis itself does not cause mental retardation. A-
D are common; hypothyroid-induced retardation is preventable by early
screening and rapid therapy. (See Chapter 37 in Nelson Textbook of
Pediatrics, 17th ed.)

Question . 24. All of the following statements about mental retardation are true except:

About 3% of the population have an IQ less than 2 standard deviations


below the mean

Severe retardation is inversely related to socioeconomic status


Explanation: Mild retardation is related to low socioeconomic status,
whereas more severe retardation is equally distributed in all such
groups. (See Chapter 37 in Nelson Textbook of Pediatrics, 17th ed.)

Up to 5% of children with mental retardation are profoundly affected

The reported incidence increases at entry to school

Language development is a first clue to mild retardation

Question . 25. Adaptive skill areas to assess in children with mental retardation include all of
the following except:

Communication

Sports
Explanation: Sports per se are not an adaptive skill but could be
categorized into leisure activities. (See Chapter 37 in Nelson Textbook
of Pediatrics, 17th ed.)

Self-care

Home living

Social skills

Community use
Question . 26. In male patients with mental retardation without an obvious etiology, the next
step in evaluation should be:

Plasma ammonia assay

Blood lead level determination

EEG

Chromosome analysis
Explanation: Fragile X syndrome manifests in males as mental
retardation, large ears, and large testes. It is relatively common. (See
Chapter 37 in Nelson Textbook of Pediatrics, 17th ed.)

Cranial CT

Question . 27. A 3-yr-old girl presents with microcephaly and mental retardation. Her mother
had a flu-like illness during the second month of pregnancy. At birth, the baby had petechiae
and hepatosplenomegaly, which have resolved. The most likely diagnosis is:

Congenital HIV infection

Congenital rubella
Explanation: Congenital rubella was once a common cause of mental
retardation. Thanks to active immunization programs, congenital
rubella is rare. (See Chapter 37 in Nelson Textbook of Pediatrics,17th
ed.)

Congenital parvovirus infection

Isoimmune neonatal thrombocytopenia

Subacute sclerosing panencephalitis

Question . 28. A 4-yr-old's concept of death may include all of the following except:

It may be reversible

It is like sleep

Dead people still eat and breathe

They understand causality


Explanation: Causality is a late developmental state of understanding. It
is important not to reinforce the immature child's belief that death is like
sleep because the child will be fearful of sleeping lest he or she may
die. (See Chapter 38 in Nelson Textbook of Pediatrics, 17th ed.)

It is a functional state
Question . 29. The fear of dying in young children is best described as fear of:

The afterlife

The unknown

Separation
Explanation: Separation from loved ones is the dominant and often only
concern of young children. (See Chapter 38 in Nelson Textbook of
Pediatrics, 17th ed.)

Transcendentalism

Existentialism

Question . 30. Perpetuating the myth of "everything is going to be all right" with a dying child
will:

Help reassure the child

Hide "bad things" from the child

Enhance an awareness of eventual death

Alleviate fears

Prevent exploration of fears


Explanation: Most chronically ill children have a sense of impending
death and need to articulate this with parents or caregivers who can
discuss fears and provide some reassurance (at least for comfort). (See
Chapter 38 in Nelson Textbook of Pediatrics, 17th ed.)

Question . 31. Giving an estimate of how long a child with a life-threatening condition will
survive will:

Be inaccurate
Explanation: Population-based statistics are poor for predicting the
time of death of an individual child. (See Chapter 38 in Nelson Textbook
of Pediatrics, 17th ed.)

Help accept the diagnosis

Avoid unnecessary expenses

Provide time to remove the child from school

Enhance communication between siblings and the child


Question . 32. All of the following statements are true except:

In the United States the most common presentation of failure to thrive


is poor growth detected in the ambulatory setting

The laboratory evaluation of children with failure to thrive is usually


helpful
Explanation: (See Chapter 36 in Nelson Textbook of Pediatrics, 17th ed.)

The history, physical examination, and observation of the parent-child


interaction usually suggest the diagnosis of failure to thrive

Failure to thrive in the first year of life regardless of cause is


particularly ominous

Approximately one third of children with psychosocial failure to thrive


are developmentally delayed and have social and emotional problems

Question . 33. All of the following statements about mental retardation (MR) are true except:

Mild MR is defined as an IQ of 80 or below


Explanation: (See Chapter 37 in Nelson Textbook of Pediatrics, 17th ed.)

In children with severe MR, a biologic cause can be identified in over


of cases

Statistically, 2.5% of the population can be expected to have MR

About 0.5% of the population have severe MR

The brains of 10-20% of individuals with severe MR are normal by


standard neuropathologic studies

Question . 34. All of the following statements are true except:

The BSID II is used to test children 1 mo-3 yr of age

The BSID II is used to identify infants with severe mental retardation

The WPPSI-R is used to test children with mental ages of 3-7 yr

The Stanford-Binet Intelligence Scale measures verbal abilities, abstract


thinking, quantitative reasoning, and adaptive functioning
Explanation: (See Chapter 37 in Nelson Textbook of Pediatrics, 17th ed.)

The WISC III is used for children who function above a mental age of 6
yr
Question . 35. All of the following statements are true except:

Surveys indicate that 15-18% of children and adolescents have some


form of chronic condition

About 6-7% of all children and adolescents have some limitation of


activity due to a chronic condition

At least 90% of children with severe long-term illnesses survive to


young adulthood

About 2-4% of children with severely long-term illnesses account for at


least 35% of child health care expenditures

Children face a relatively small number of common chronic conditions


and a few rare diseases
Explanation: (See Chapter 37 in Nelson Textbook of Pediatrics, 17th ed.)

Question . 36. John is 12 yr old and has Down syndrome. He has recently transitioned from an
elementary school that used an inclusion model to a middle school program that continues to
focus on reading and math. John has been spending more time in his room and less time with
the family. He is less lively and more irritable. The most likely diagnosis is:

Dementia

Hypothyroidism

Adjustment disorder
Explanation: (See Chapter 37 in Nelson Textbook of Pediatrics, 17th ed.)

Depression

Atlantoaxial subluxation
Question . 37. Of the following causes of severe mental retardation, the most common is:

Inborn errors of metabolism

Developmental brain anomalies

Perinatal causes

Chromosomal abnormalities
Explanation: (See Chapter 37 in Nelson Textbook of Pediatrics, 17th ed.)

Postnatal causes
Question . 38. The most common reason for people with mental retardation to be placed out of
the home is:

Single-parent family

Greater degree of mental retardation

Greater intensity of medical needs

Lack of an appropriate school, necessitating residential placement

Behavioral disturbance
Explanation: (See Chapter 37 in Nelson Textbook of Pediatrics, 17th ed.)

Question . 39. In addition to the deficits in cognition and onset before age 18 yr, the diagnosis
of mental retardation requires deficits or impairments in:

Adaptive behavior
Explanation: (See Chapter 37 in Nelson Textbook of Pediatrics, 17th ed.)

Stereotypic behavior

Self-stimulatory behavior

Play

Moor behavior

Question . 40. Jane is an 18-mo-old girl who presents because of delayed walking. Findings on
the motor examination are normal, save for mild, diffuse hypotonia. Further evaluation should
include:

Chromosome analysis

Assessment of language skills


Explanation: (See Chapter 37 in Nelson Textbook of Pediatrics, 17th ed.)

EMG and nerve conduction studies

MRI

Audiologic evaluation
Question . 41. All of the following statements are true except:

Provision of respite services is especially important to families caring


for children with complex chronic conditions over prolonged periods of
time

Good end-of-life care cannot be effectively carried out in a hospital


setting
Explanation: (See Chapter 38 in Nelson Textbook of Pediatrics, 17th ed.)

Children's fear of death is centered on the concrete fear of being


separated from parents and other loved ones

In some families and cultures, truth telling and autonomy are much less
valued than family integrity

Many dying children do not have their pain successfully treated

Question . 42. All of the following statements are true except:

Physicians should ask parents of the patient about their economic


circumstances

The medical problems of children of migrant farm workers are similar to


those of children of homeless families

By age 20-30 yr, few children who were at special risk have made
moderate success of their lives
Explanation: (See Chapter 39 in Nelson Textbook of Pediatrics, 17th ed.)

Most states authorize treatment of homeless youth

Recurrent otitis media is an especially frequent problem among Native


American children
Nutrition

Question . 1. The breast-fed infant of a mother who is a strict vegan may experience deficiency
of which of the following vitamins if the mother is not receiving supplements of the vitamin?

B6

B12
Explanation: A strict vegan diet contains no eggs, meat, or milk products
and is thus deficient in vitamin B12. (See Chapters 41, 42, and 44 in
Nelson Textbook of Pediatrics, 17th ed.)

Folate

Biotin

Question . 2. The best source of iron for 1-mo-old infants is:

Iron-fortified cereals

Yellow vegetables

Fruits

Breast milk
Explanation: Although breast milk contains relatively less iron by weight,
the iron is more bioavailable than the iron in cereals. Fruits, yellow
vegetables, and cow's milk are poor sources of iron. (See Chapters 40
and 41 in Nelson Textbook of Pediatrics, 17th ed.)

2% low-fat cow's milk

Question . 3. A 4-mo-old child with vitamin D deficiency rickets would be expected to show all of
the following except:

Craniotabes

Bowleg
Explanation: In rickets, parathyroid hormone level is elevated, which
results in low serum phosphate levels. Low serum phosphate levels
result in abnormal osteoblastic activity, which may result in craniotabes
and a rachitic rosary with enlargement at the costochondral junctions.
Even though osteoid of the legs is uncalcified, bowing does not occur
until weight is borne on the legs. (See Chapter 44 in Nelson Textbook of
Pediatrics, 17th ed.)

Rosary
Low serum phosphate levels

High alkaline phosphatase levels

Question . 4. The estimated average requirement (EAR) of a nutrient is best defined as:

A dietary reference index

The recommended dietary allowance

Age- and gender-specific to meet the needs of 50% of persons


Explanation: The EAR is one form of nutrient evaluation. The RDA
(answer B) is the daily dietary intake sufficient to meet 97-98% of
individual nutrient needs in age and gender groups. (See Chapter 40 in
Nelson Textbook of Pediatrics, 17th ed.)

The range encompassing the lower and upper limits of a nutrient

A daily average calculated once each year

Question . 5. The AI (adequate intake) in infants is estimated from:

Bomb calorimetry

The EAR

The RDA

The intake of nutrients from human milk


Explanation: The AI is one component of the DRI (dietary reference
intakes). The DRI also considers the EAR, RDA, and tolerable upper level
(UL). (See Chapter 40 in Nelson Textbook of Pediatrics, 17th ed.)

Intakes that will prevent deferrable nutrient deficiencies

Question . 6. All of the following are advantages of breast-feeding except:

Reduced incidence of allergy

Reduced incidence of otitis media

Reduced incidence of colic

Increased psychologic comfort

Vitamin K content
Explanation: Vitamin K must be given (intramuscularly at birth) to all
infants. Breast-fed infants whose diet is not supplemented with vitamin K
are at risk for bleeding. (See Chapter 41 in Nelson Textbook of Pediatrics,
17th ed.)

Utility for preterm infants weighing less than 2000 g

Question . 7. Problems associated with breast-feeding include all of the following except:

Less than optimal nutrients for infants <1000 g

Vitamin K content

Transmission of live viruses

Hyperbilirubinemia

Contraindication in erythroblastosis fetalis


Explanation: Antibodies in mother's milk are inactivated in the infant's
intestines and do not contribute to intravascular hemolysis. (See Chapter
41 in Nelson Textbook of Pediatrics, 17th ed

Question . 8. Atypical features of infant colic include all of the following except:

Fever

Onset in the first week of life

Onset at age 6 mo

Sudden onset

Crying mainly in the early morning


Explanation: Colic usually recurs in the early evening. (See Chapter 41 in
Nelson Textbook of Pediatrics, 17th ed.)
Question . 9. All of the following observations regarding infant feeding are true except:

Self-feeding with a spoon at age 12 mo should be encouraged

Infants will select a balanced diet

Consistent rejection of one food group should suggest food allergy

Infants should be put to bed with a bottle of milk


Explanation: Baby bottle caries syndrome is a significant problem
because the milk bathes the teeth, setting up multiple caries. (See
Chapter 41 in Nelson Textbook of Pediatrics, 17th ed.)

The likes and dislikes of infant tastes should be respected


Question . 10. All of the following are clinical manifestations of kwashiorkor except:

The presence of edema

Rash in sun-exposed areas


Explanation: The rash of kwashiorkor is in areas of irritation. Sun-
exposed dermatitis is typical of pellagra. (See Chapter 42 in Nelson
Textbook of Pediatrics, 17th ed.)

Hypochromotrichia

Muscle weakness

An increased susceptibility to infection

Question . 11. All of the following are laboratory manifestations of kwashiorkor except:

Persistent ketonuria
Explanation: Ketonuria is present early but does not persist into the later
stages. (See Chapter 42 in Nelson Textbook of Pediatrics, 17th ed.)

Hypoalbuminemia

Hypoglycemia

Potassium deficiency

Low serum amylase levels

Question . 12. All of the following statements about obesity in children are true except:

Obese children eat more junk food


Explanation: Obese children do not necessarily eat more food or more
junk food than leaner children. (See Chapter 43 in Nelson Textbook of
Pediatrics, 17th ed.)

Single-gene disorders are rare causes of obesity

Obesity may be associated with insulin resistance

The highest prevalence of obesity in the United States is in the Northeast

Menarche may be earlier in obese girls

Question . 13. Complications of obesity in childhood include all of the following except:

Angina
Explanation: Although hyperlipidemia secondary to obesity may be
present, coronary artery disease does not manifest in children. (See
Chapter 43 in Nelson Textbook of Pediatrics, 17th ed.)

Blount disease

Slipped capital femoral epiphysis

Sleep apnea

Glucose intolerance

Question . 14. Physical features of vitamin D deficiency rickets include all of the following
except:

Bitot spots
Explanation: Bitot spots are seen in vitamin A deficiency (dry plaques on
the bulbar conjunctiva). (See Chapter 44 in Nelson Textbook of
Pediatrics, 17th ed.)

Craniotabes

Enlargement of the costochondral junctions

Thickening of the ankles and wrists

Large anterior fontanel

Bowleg

Question . 15. Clinical features of vitamin E deficiency include all of the following except:

Cerebellar ataxia

Muscle weakness

Peripheral neuropathy

Hemolysis

Hepatosplenomegaly
Explanation: A-C often occur in patients with malabsorption. D has been
reported in premature infants. (See Chapter 44 in Nelson Textbook of
Pediatrics, 17th ed.)
Question . 16. The pediatric patient with obesity typically presents with:

Tall stature
Explanation: (See Chapter 43 in Nelson Textbook of Pediatrics, 17thed.)

Delayed bone age

Negative family history of obesity

Delayed puberty

Elevated serum cholesterol

Question . 17. The group that has exhibited the most striking increase in the prevalence of
obesity in the United States is:

Caucasian males

Caucasian females

African-American and Hispanic males

African-American and Hispanic females


Explanation: (See Chapter 43 in Nelson Textbook of Pediatrics, 17thed.)

Asian-American males and females

Question . 18. The risk of developing obesity in adulthood correlates negatively with:

High birthweight

Number of obese parents

Months of breast-feeding
Explanation: (See Chapter 43 in Nelson Textbook of Pediatrics, 17thed.)

Hours of television viewing

Weight during adolescence


Pathophysiology of Body Fluids and Fluid Therapy

Question . 1. Diabetes insipidus may be due to all of the following except:

Pituitary adenoma

Renal epithelial ADH reception defect

Hypokalemia

Hypercalcemia

Adrenal deficiency
Explanation: Adrenal deficiency may cause renal salt wasting and
usually does not affect free water excretion. (See Chapter 45.3 in Nelson
Textbook of Pediatrics, 17th ed.)

Question . 2. A 1-mo-old boy presents with severe failure to thrive, emesis, and a temperature of
41C. Serum electrolyte measurements reveal a sodium level of 185 mEq/L, and the urine
specific gravity is 1001. The most likely diagnosis is:

Adrenal insufficiency

Salt poisoning

Hypernatremic dehydration

Malignant hyperthermia

Nephrogenic diabetes insipidus


Explanation: Nephrogenic diabetes insipidus is a sex-linked recessive
disorder due to deficient binding of ADH to the renal tubular cell.
Exogenous administration of ADH is therefore ineffective. (See Chapter
45.3 in Nelson Textbook of Pediatrics, 17th ed.)
Question . 3. A well-grown 6-mo-old presents with a tonic-clonic seizure lasting 30 min. The
child is found to be hypothermic and remains lethargic. The diet history reveals that the mother
is a participant in the WIC program. Because it is the end of the month, she has begun to dilute
the remaining formula with water as there is not enough to last until she receives her next
allotment of formula next week. The most likely diagnosis is:

Hypocalcemia

Hyponatremia
Explanation: Hyponatremia due to feeding diluted formula or excessive
amounts of sodium-free fluids (especially water) is relatively and
unfortunately common among poor families who run out of formula. (See
Chapter 46 in Nelson Textbook of Pediatrics, 17th ed.)

Hypoglycemia

Hypernatremia

Hypokalemia

Question . 4. Hyperkalemia may be associated with all of the following except:

Succinylcholine use

Burns

Trauma

Chemotherapy

Metabolic alkalosis
Explanation: Metabolic alkalosis produces hypokalemia. (See Chapters
45.4 and 45.8 in Nelson Textbook of Pediatrics,17th ed.)

Digitalis toxicity

Uremia

Question . 5. A normal anion gap acidosis is most likely to be due to:

Diabetes mellitus

Renal tubular acidosis


Explanation: Renal tubular acidosis with renal bicarbonate loss and
diarrhea-induced stool losses of bicarbonate are the common causes of
a normal anion gap acidosis. (See Chapter 45.8 in Nelson Textbook of
Pediatrics,17th ed.)

Nephrotic syndrome

Uremia

Shock
Question . 6. A 10-mo-old infant presents with vomiting and diarrhea, tachycardia, normal blood
pressure, dry mucous membranes, a capillary refill time of 2 sec, deep respirations, and
irritability. The percent dehydration for this patient is:

0-3%

3-5%

6-9%
Explanation: Dehydration of 6-9% represents moderate dehydration and
early shock. Tachycardia reflects the intravascular volume loss, and
deep respirations represent the pulmonary response to metabolic
acidosis. (See Chapters 47 and 48 in Nelson Textbook of Pediatrics, 17th
ed.)

10-12%

12-15%
Question . 7. A serious complication of the treatment of hypernatremic dehydration is:

Cerebral thrombosis

Cerebral edema
Explanation: Cerebral edema occurs if free water is given in excessive
amounts, if the serum sodium falls more than 10 mEq/L/day, and if
idiogenic osmoles remain in neurons during rehydration. Cerebral
thrombosis may occur before therapy is started and may be associated
with inherited hypercoagulable conditions. (See Chapters 45.3 and 47 in
Nelson Textbook of Pediatrics, 17th ed.)

Hyperchloremia

Hypoglycemia

None of the above

Question . 8. The best method to reduce the potassium level during hyperkalemia, by reducing
the body burden of potassium, is:

Sodium bicarbonate infusion

Glucose and insulin infusion

Calcium infusion

Albuterol aerosol

Kayexalate enema
Explanation: Kayexalate, a potassium-binding resin, and dialysis are the
only methods to remove potassium from the body. Other methods shift
potassium from the extracellular to the intracellular space. (See Chapter
45.4 in Nelson Textbook of Pediatrics, 17th ed.)
Question . 9. The finding of marked metabolic alkalosis with acidic urine indicates:

Marked sodium depletion

Marked potassium depletion


Explanation: Marked potassium depletion is more likely, although
laboratory error is possible. When in doubt, repeat laboratory tests, but
such a repeat in this case would confirm the finding. (See Chapter 45.4 in
Nelson Textbook of Pediatrics, 17th ed.)

Hyperventilation

Diabetes mellitus

Laboratory error

Question . 10. From the following list, choose the route(s) by which insensible water loss may
occur.
1. Sweat
2. Fecal loss
3. Evaporative loss from skin
4. Respiratory water loss
5. Obligate water for urinary solute excretion

1 and 3

1, 2, and 3

3 only

3 and 4
Explanation: Insensible water losses usually occur independently of total
body water homeostasis. (See Chapter 45.2 in Nelson Textbook of
Pediatrics, 17th ed.)

2 and 5

Question . 11. Which of the following drugs or agents may inhibit antidiuretic hormone release?

Demerol

Barbiturates

Alcohol
Explanation: All the others may stimulate ADH release. (See Chapter 45.3
in Nelson Textbook of Pediatrics,17th ed.)

Nicotine

-Adrenergic drugs
Question . 12. Hypernatremia may be induced by all of the following except:

Hyperglycemia
Explanation: Indeed, hyperglycemia may produce pseudohyponatremia.
(See Chapter 45.3 in Nelson Textbook of Pediatrics, 17th ed.)

Adipsia

Insufficient breast-feeding

Gastroenteritis

Nephrogenic diabetes insipidus

Question . 13. The most common cause of nutritional hyponatremia is:

Salt substitutes

Low-salt diets

The WIC syndrome


Explanation: The WIC syndrome in families receiving formula from the
Women's, Infants', and Children's nutritional supplementation program
occurs when families run out of formula toward the end of the month and
begin to dilute the formula with water. Another common cause is feeding
excess amounts of pure water in general. (See Chapter 45.3 in Nelson
Textbook of Pediatrics, 17th ed.)

Furosemide (Lasix) therapy

Vegan diets

Question . 14. Manifestations of hyperkalemia include all of the following except:

Paresthesias

Weakness

Paralysis

Wide QRS complex

Tetany
Explanation: A-D are noted in hyperkalemia. The first ECG change is
peak T waves. Lengthening of the P-R interval and QRS complex occurs
later. (See Chapter 45.4 in Nelson Textbook of Pediatrics, 17th ed.)
Question . 15. Potential causes of hyperkalemia include all of the following except:

Succinylcholine

Digitalis toxicity

Acute renal failure

Albuterol overdose
Explanation: -Agonists often cause hypokalemia. (See Chapter 45.4 in
Nelson Textbook of Pediatrics, 17th ed.)

Captopril overdose

Question . 16. An increased anion gap occurs in all of the following except:

Diabetic ketoacidosis

Renal tubular acidosis


Explanation: RTA produces a non-anionic gap acidosis due to loss of
bicarbonate by the kidney and not due to net gain of new acid in the
circulation. (See Chapter 45.8 in Nelson Textbook of Pediatrics, 17th ed.)

Salicylate poisoning

Methylmalonicacidemia

Ethylene glycol poisoning

Question . 17. A preterm infant born to a mother with severe preeclampsia is noted to be
hypotonic and apneic in the delivery room. After resuscitation and stabilization, she remains
hypotonic with decreased deep tendon reflexes in the arms and knees. The mother's treatment
included hydralazine, magnesium sulfate, and indomethacin. The laboratory evaluation of this
patient should include:
1. Serum calcium determination
2. Arterial blood gas analysis
3. Serum magnesium determination
4. CBC
5. Anion gap measurement

1 and 3

2 only

1, 2, and 3
Explanation: 1, 2, and 3 would be helpful in the management of these
immediate problems. A CBC would be of some use if anemia or infection
were suspected. (See Chapters 45.5 and 45.6 in Nelson Textbook of
Pediatrics, 17th ed.)

2 and 4

2, 3, and 5
Question . 18. The serum magnesium level for the patient in Question 17 is 6.5 mg/dL. From the
following list, select the most appropriate next step(s) in treatment.
1. Continue mechanical ventilation
2. Infuse normal saline
3. Add calcium to the intravenous solution
4. Begin chelation therapy
5. Administer KCl

1, 2, and 3
Explanation: 1, 2, and 3 are correct, although with supportive care and
time, most patients do quite well. (See Chapters 45.5 and 45.6 in Nelson
Textbook of Pediatrics, 17th ed.)

3 only

4 only

1 and 5

2 and 4

Question . 19. Possible consequences of hypophosphatemia include all of the following except:

Hypocalcemia
Explanation: Indeed, hypercalcemia is more common as a compensatory
mechanism to release phosphate from bone. (See Chapter 45.7 in Nelson
Textbook of Pediatrics, 17th ed.)

Hemolysis

Rhabdomyolysis

Paresthesias

Confusion

You might also like